Inégalité entre deux intégrales

Bonjour à tous !
Je sèche actuellement sur une inégalité à montrer entre deux intégrales. L'énoncé est le suivant.

Soit $f \in \mathcal{C}^1 ([0, 1],\mathbb{R})$ telle que $f(0) = f(1) = 0$. Démontrer que : $$

\int_0^1 \frac{f(t)^2}{t(1-t)} \mathrm{d}t \leq \frac{1}{2} \int_0^1 {f'}^2 .

$$ J'ai réussi à montrer l'existence des deux intégrales (pas de problème là-dessus), mais je n'arrive pas à montrer l'inégalité voulue. J'ai essayé des intégrations par parties, l'inégalité de Cauchy-Schwarz, montrer le résultat sur un segment strictement inclus dans $[0, 1]$ pour ensuite faire tendre les bornes vers $0$ et $1$, rien à faire... j'arrive à des résultats "proches" (c'est-à-dire qui ont une vague ressemblance avec le résultat voulu), mais qui n'aboutissent jamais à l'inégalité souhaitée (et pourtant ça n'a pas l'air bien difficile). Quelqu'un voit-il comment s'en sortir ?

Je vous remercie d'avance pour votre aide, et prenez soin de vous (en espérant que l'épidémie prenne fin assez vite) !

Réponses

  • J'ai une idée mais je n'ai pas encore testé. Écrire
    \[\frac{f(t)^2}{t(1-t)}=\frac{f(t)-f(0)}{t}\times \frac{f(t)-f(1)}{1-t}=\left(\int_0^1 f'(t u)du\right) \times \left(-\int_0^1 f'((1-t)v+t)dv\right)\] puis Fubini... et prier pour que ça colle.
  • Bonjour
    En coupant l'intégrale en 2 et un changement de variable $x\mapsto 1-x$ (pour l'intégrale de 1/ 2 à 1)
    il suffit de montrer que $$\int_{0}^{1/2} \dfrac{f(x)^2}{x(1-x)} dx \leq \dfrac{1}{4} \int_{0}^{1/2} f'(x) ^2 dx.

    $$ Par C-S avec $f'(x)=\int_0 ^u f'(u) du$ on obtient $$

    \int_{0}^{1/2} \dfrac{f(x)^2}{x(1-x)} dx \leq \int_{0}^{1/2}\Big( \dfrac{1}{1-x}\int_0 ^x f'(u)^2 du \Big) dx ,

    $$ et on finit encore une fois avec C-S.
     
  • Bonsoir

    En écrivant $f(t)=(t-t^2)\: g(t) \:$ avec $g\:$ de classe $\mathcal C^1$ sur $]0;1[$ et $\:\: \displaystyle \lim _{t\to 0^+}(t - t^2)\: g'(t) = \lim_{t \to 1^-} (t-t^2) \:g'(t) =0\:$, on aboutit à : $$
    I:= \int _0^1 f'^2 -2\int _0^1 \dfrac {f(t)^2}{t-t^2} \:\mathrm dt = \int_0^1 g'(t) ^2 (t-t^2) ^2 \:\mathrm dt \geqslant 0.$$
    L'égalité est donc réalisée lorsque $f(t) = t-t^2.$
    Les détails:
    $\begin {align*} I= & \displaystyle \int _0^1 \left[ \left( g(t) (1-2t)+ g'(t) (t-t^2)\right)^2- 2g(t)^2 (t-t^2) \right] \: \mathrm dt \\ = & \int _0 ^1 (6t^2 - 6t +1) g(t)^2 \:\mathrm dt + \int _0 ^12 (2t^3 -3t^2 +t)g(t)g'(t) \:\mathrm dt + \int _0^1 (t-t^2)^2 g'(t)^2 \: \mathrm dt \\ =& \int _0^1 (6t^2 - 6t +t) g(t) ^2 \: \mathrm dt + \left [ g(t)^2 (2t^3 -3t^2 +t)\right] _0 ^1 - \int _0 ^1 (6t^2 -6t+1) g(t)^2 \:\mathrm dt+ \int_0 ^1 (t-t^2) ^2 g'(t) ^2 \:\mathrm dt \\ = & \int _0 ^1 (t-t^2)^2 g'(t)^2 \:\mathrm dt\end{align*}$
  • @LOU16 Je dois être rouillé, mais je ne comprends pas pourquoi vous pouvez écrire "$f(t)=(t-t^2)\: g(t) \:$ avec $g\:$ de classe $\mathcal C^1$ sur $]0;1[$ et $\displaystyle \lim _{t\to 0^+}(t - t^2)\: g'(t) =\lim_{t \to 1^-} (t-t^2) \:g'(t) =0\:$". La partie "$f(t)=(t-t^2)\: g(t) \:$ avec $g\:$ de classe $\mathcal C^1$ sur $]0;1[$" ne me pose pas de problème mais pourquoi a-t-on "$\displaystyle \lim _{t\to 0^+}(t - t^2)\: g'(t) =\lim_{t \to 1^-} (t-t^2) \:g'(t) =0\:$"? Est-ce que $g'$ est bornée au voisinage de $0$ et au voisinage de $1$? Si oui, pourquoi?

    Mais déjà merci pour vos réponses!
  • @bd2017 Il y a deux points que je ne comprends pas dans vos explications. D'abord, lorsqu'on fait le changement de variable que vous indiquez, on se retrouve bien avec les bornes $0$ et $1/2$, mais dans l'expression de l'intégrale il y a $f(1-x)^2$ au lieu de $f(x)^2$. Pourriez-vous détailler?

    Ensuite, lorsque vous dites que l'on finit avec Cauchy-Schwarz, j'ai essayé de mon côté mais je trouve un facteur $\frac{1}{\sqrt{2}}$ au lieu de $\frac{1}{4}$. Pourriez-vous détailler également ce point?

    Merci d'avance!
  • Bonjour,
    @Adrien
    \begin{align*}\forall t \in ]0;1[,\qquad
    (t-t^2)g'(t) &= f'(t) +\left( \dfrac t{1-t} -\dfrac {1 -t}t \right) f(t) ,\\
    \lim _{t\to 0 ^+} (t-t^2) g'(t) &= f'(0) - \lim_{t\to 0^+} \dfrac {f(t) -f(0)}t =0,\\
    \lim_{t\to 1^-} (t-t^2)g'(t)& = f'(1) - \lim _{t\to 1^-} \dfrac {f(t) - f(1)} {t-1} = 0.
    \end{align*}
  • @LOU16 d'accord merci beaucoup!

    @bd2017 pour la première des deux questions que je vous ai posées je crois que j'ai une idée mais je ne suis pas encore certain que c'est juste donc je veux bien des détails. Et pour la deuxième question je ne comprends toujours pas (si vous pouviez détailler l'utilisation de Cauchy-Schwarz et les égalités qui mènent à la conclusion ça m'aiderait). Au passage, vous avez écrit "$f'(x)=\int_0 ^u f'(u) du$". Vous vouliez sans doute dire "$f(x)=\int_0 ^x f'(u) du$" (pas de "prime" devant le premier $f$ et l'intégrale va de $0$ à $x$ et pas de $0$ à $u$ ;-) ).
  • Bonjour
    Bon ça ne va pas ce que j'ai fait. ça donne bien une majoration mais la constante n'est pas optimale.
    Il faut pour l'instant rester sur la démo de Lou16 qui est intéressante et qui donne la majoration optimale.
    Quelle est l'idée sous-jacente ?
    Je cherche encore un peu pour avoir une démo dans l'esprit de C-S.
     
  • @bd2017 d'accord. Du coup la démonstration de LOU16 montre que la constante $\frac{1}{2}$ est optimale? Je ne vois pas en quoi LOU16 a montré qu'il n'y avait pas de meilleure constante. Et que voulez-vous dire par "Quelle est l'idée sous-jacente?"?
  • Renbonjour
    Dans ce genre d'inégalité (ici la constante est 1/2), il est parfois utile de savoir si la constante est la meilleure.
    C'est le cas ici en prenant l'exemple f(x)=x(1-x).
    Quand je parle de l'idée sous-jacente de la démo de @Lou16, c'est en fait l'idée de départ qui l'a amené à poser la fonction auxiliaire g(t).
     
  • @bd2017 ok pour l'optimalité de la constante. Pour ce qui est de l'idée sous-jacente, je dirais juste comme on divise par $t(1-t)=t-t^2$ dans l'intégrale, on va poser $f(t)=(t-t^2)g(t)$ pour faire disparaître le dénominateur dans l'intégrale. D'ailleurs c'est $f^2$ et pas $f$ qu'on divise par $t(1-t)$ donc je ne vois pas pourquoi on n'aurait pas pu poser $f(t)= \sqrt{t-t^2} g(t)$ (ce qui ferait aussi disparaître le dénominateur). Après pour moi la seule raison d'avoir posé $f(t)=(t-t^2)g(t)$ c'est "parce qu'en faisant les calculs on se rend compte que ça marche". Je me doute bien que vous attendez sans doute une idée plus profonde mais je ne vois pas laquelle...??
  • Ces inégalités de type Wirtinger https://dergipark.org.tr/en/download/article-file/698009 peuvent se démontrer de plusieurs façons (Fubini, Parseval, etc).
  • Bonjour,

    Montrer que, pour toute fonction $f$ de $\displaystyle [0,1]$ vers $\displaystyle \R$, dérivable sur $\displaystyle [0,1]$, telle que $\displaystyle f(0)=f(1) =0$ : $\displaystyle \int_0^1 {f(t)^2 \over t(1-t)}dt \leq {1 \over 2} \int_0^1 f'(t)^2 dt.$

    Voici :

    $\bullet $ On note $g$ la fonction de $\displaystyle ]0,1[$ vers $\displaystyle \R$, dérivable sur $\displaystyle ]0,1[$ : $\displaystyle g: t \mapsto {1 \over t} - {1 \over 1-t}.$
    Pour tout $\displaystyle t \in ]0,1[$, on calcule $\displaystyle g'(t) + g(t)^2 = -{2 \over t(1-t)}.$

    $\bullet $ La fonction $\displaystyle t \mapsto g(t) f(t)$ est intégrable sur $\displaystyle [0,1].$ En effet, $\displaystyle g(t) f(t) = {f(t) \over t} - {f(t) \over 1-t} ={f(t) -f(0)\over t-0} - {f(t) \over 1-t} \to f'(0)-f(0), (t \to 0)$ et $\displaystyle g(t) f(t) = {f(t) \over t} - {f(t) \over 1-t} = {f(t) \over t} + {f(t)-f(1) \over t-1} \to f(1) + f'(1), (t \to 1).$

    $\bullet $ La fonction $\displaystyle t \mapsto g'(t) f(t)^2$ est intégrable sur $\displaystyle [0,1].$ En effet, $\displaystyle g'(t) f(t)^2 = -{f(t)^2 \over t^2} -{f(t)^2 \over (1-t)^2} = -({f(t) - f(0) \over t-0})^2 -{f(t)^2 \over (1-t)^2} \to -f'(0)^2-f(0)^2, (t \to 0)$ et $\displaystyle g'(t) f(t)^2 = -{f(t)^2 \over t^2} -{f(t)^2 \over (1-t)^2} = -{f(t)^2 \over t^2} -({f(t)-f(1) \over t-1})^2 \to -f(1)^2 - f'(1)^2, (t \to 1).$

    $\bullet $ On calcule tout de go, avec une intégration par parties :
    $\displaystyle \int_0^1 (f'(t) + g(t) f(t))^2 dt = \int_0^1 (f'(t)^2 + g(t)^2 f(t)^2 + 2 g(t) f(t) f'(t)) dt = \\\displaystyle =\int_0^1 f'(t)^2 dt + \int_0^1 g(t)^2 f(t)^2 dt - g(t) f(t)^2 |_0^1 + \int_0^1 g'(t) f(t)^2 dt = \int_0^1 f'(t)^2 dt + \int_0^1 (g'(t) + g(t)^2) f(t)^2 dt \geq 0 (\star)$

    Voilà !

    J'ai bien sûr commencé par écrire $(\star)$ puis résolu avec un peu de recherche, pour tout $\displaystyle t \in ]0,1[$, $\displaystyle g'(t) + g^2(t) = -{2 \over t(1-t)}.$
  • Bonjour à tous,

    Désolé pour ma réponse très tardive, mais j'ai attrapé le virus (j'étais plus au lit que sur mon ordi, et même maintenant je suis pas encore remis) :-( . Mais ça va un peu mieux!

    @YvesM du coup ça marcherait pour $f$ seulement supposée dérivable? Et quand vous dîtes dérivable sur $[0,1]$, je suppose que ça veut dire dérivable à droite en $0$ et à gauche en $1$? Sinon un point que je ne comprends pas dans votre démonstration est l'intégration par parties. Pour moi les fonctions doivent être $\mathcal{C}^1$. Pourriez-vous détailler cette IPP? En particulier, quelles sont les fonctions $\mathcal{C}^1$ que vous utilisez (sachant que $f$ est seulement dérivable)?

    @bd2017 je ne vois toujours pas quelle est l'idée derrière la démonstration de LOU16; pourquoi penser à poser une fonction $g$ auxiliaire et pourquoi $g$ est-elle ainsi choisie?

    Merci pour les réponses que vous avez déjà apportées, et encore désolé pour mon retour tardif!
  • Bonjour,

    Une fonction dérivable sur un ouvert est continue sur cet ouvert. Non ?

    Ce n’est pas moi mais l’énoncé qui précise que la fonction $f$ est dérivable et de dérivée continue sur $[0,1]$. Non ?

    L’intégration par partie porte sur $g$ et sur $f.f’$ : $\int 2 g.f.f’=g. f^2-\int g’ f^2.$

    La fonction $g.f^2$ est le produit de deux fonctions continues, dérivables, et de dérivées continues sur $]0,1[$. J’ai également montré que les limites en $0$ et en $1$ existent... donc toutes les intégrales dans cette intégration par parties existent. Non ?
Connectez-vous ou Inscrivez-vous pour répondre.